(One intermediate revision by the same user not shown)
Line 27: Line 27:
 
=Solution=
 
=Solution=
 
:'''Click [[ECE_PhD_QE_FO_2013_Problem1.1|here]] to view student [[ECE_PhD_QE_FO_2013_Problem1.1|answers and discussions]]'''
 
:'''Click [[ECE_PhD_QE_FO_2013_Problem1.1|here]] to view student [[ECE_PhD_QE_FO_2013_Problem1.1|answers and discussions]]'''
<math>
 
\begin{equation*}
 
\left.\begin{aligned}
 
\nabla\cdot \bar{B}&=0\\
 
\oint_S \bar{B}\cdot d\bar{S}&=0
 
\end{aligned}\right\}
 
\longrightarrow \Phi=\oint_S \bar{B}\cdot d\bar{S} \Longrightarrow  \boxed{ \Phi=\oint_S \bar{B}\cdot d\bar{S}=0}
 
\end{equation*}
 
</math>
 
 
The magnetic flux through this closed surface is <math>\Phi</math>
 
 
<math>
 
\begin{equation*}
 
\boxed{\Phi=0}
 
\end{equation*}
 
</math>
 
  
 
==Question 2==
 
==Question 2==
Line 49: Line 32:
  
 
=Solution=
 
=Solution=
[[Image:cube.png|Alt text|300x300px]]
+
:'''Click [[ECE_PhD_QE_FO_2013_Problem2.1|here]] to view student [[ECE_PhD_QE_FO_2013_Problem2.1|answers and discussions]]'''
 
+
<math>
+
\begin{align*}
+
\nabla \cdot \bar{D} &= \rho \\
+
\quad (\frac{\partial}{\partial x}\hat{x}+\frac{\partial}{\partial y}\hat{y}+\frac{\partial}{\partial z}\hat{z})\cdot(2\hat{x})&=\rho \\
+
\frac{\partial}{\partial x}(2)&=0=\rho \quad \text{(no charge)}
+
\end{align*}
+
</math>
+
 
+
Also:
+
 
+
<math>
+
\begin{align*}
+
\oint \bar{D}\cdot d\bar{S}&=Q\\
+
&=\int2(dS_x)+\int2(-dS_x)=2-2=\boxed{0}
+
\end{align*}
+
</math>
+
  
 
==Question 3==
 
==Question 3==
Line 74: Line 40:
  
 
=Solution=
 
=Solution=
Using superposition <br/>
+
:'''Click [[ECE_PhD_QE_FO_2013_Problem3.1|here]] to view student [[ECE_PhD_QE_FO_2013_Problem3.1|answers and discussions]]'''
In the left cylinder <br/>
+
<math>
+
\begin{equation*}
+
\nabla\times \bar{H}=\bar{J} \longrightarrow \oint \bar{H}\cdot d\bar{l}=\int_S\bar{J}\cdot d\bar{S}
+
\qquad \left\{ \begin{aligned}
+
dl&=dr\hat{r}+rd\phi\hat{\phi}+dz\hat{z}\\
+
d\bar{S}_z&=rd\phi dr\hat{z}
+
\end{aligned} \right.
+
\end{equation*}
+
</math>
+
<br/>
+
<math>
+
\begin{align*}
+
\int_0^{2\pi}H_{\phi}(rd\phi)&=\int_0^r\int_0^{2\pi} J_0(r'd\phi dr')\\
+
H_{\phi}(2\pi r)&=2\pi J_0(\frac{r^2}{2}) \\
+
& \boxed{\bar{H}=\frac{J_0r}{2}\hat{\phi}}
+
\end{align*}
+
</math>
+
<br/>
+
[[Image:cil.png|Alt text|300x300px]]
+
<br/>
+
<math>
+
\begin{align*}
+
\text{Transform to cartesian:}&\left\{\begin{aligned}
+
r&=\sqrt{x^2+y^2}\\
+
\hat{\phi}&=-sin\phi\hat{x}+cos\phi\hat{y}\\
+
&=(\frac{-y}{\sqrt{x^2+y^2}})\hat{x}+(\frac{x}{\sqrt{x^2+y^2}})\hat{y}
+
\end{aligned}\right. \\
+
& \boxed{\bar{H}_L=\frac{J_0}{2}\left[-y\hat{x}+x\hat{y}\right]}
+
\end{align*}
+
</math>
+
 
+
In the right cilinder <br/>
+
<math>
+
\begin{align*}
+
&\bar{H}_R=\frac{-J_0}{2}\left[-y'\hat{x'}+x'\hat{y'}\right]&
+
\left\{
+
\begin{aligned}
+
x'&=x-3\\
+
y'&=y\\
+
\hat{x}'&=\hat{x}\\
+
\hat{y}'&=\hat{y}
+
\end{aligned}
+
\right.\\
+
&\boxed{\bar{H}_R=\frac{-J_0}{2}\left[-y\hat{x}+(x-3)\hat{y}\right]}&
+
\end{align*}
+
</math>
+
 
+
<math>
+
\begin{equation*}
+
\boxed{\bar{H}_T=\bar{H}_L+\bar{H}_R=\frac{3J_0}{2}\hat{y}}
+
\end{equation*}
+
</math>
+
 
+
 
+
 
+
 
+
 
+
==Question==
+
'''Part 1. '''
+
 
+
Consider <math class="inline">n</math> independent flips of a coin having probability <math class="inline">p</math> of landing on heads. Say that a changeover occurs whenever an outcome differs from the one preceding it. For instance, if <math class="inline">n=5</math> and the sequence <math class="inline">HHTHT</math> is observed, then there are 3 changeovers. Find the expected number of changeovers for <math class="inline">n</math> flips. ''Hint'': Express the number of changeovers as a sum of Bernoulli random variables.
+
 
+
:'''Click [[ECE_PhD_QE_CNSIP_2013_Problem1.1|here]] to view student [[ECE_PhD_QE_CNSIP_2013_Problem1.1|answers and discussions]]'''
+
----
+
'''Part 2.'''
+
 
+
Let <math>X_1,X_2,...</math> be a sequence of jointly Gaussian random variables with covariance
+
 
+
<math>Cov(X_i,X_j) = \left\{ \begin{array}{ll}
+
{\sigma}^2, & i=j\\
+
\rho{\sigma}^2, & |i-j|=1\\
+
0, & otherwise
+
  \end{array} \right.</math>
+
 
+
Suppose we take 2 consecutive samples from this sequence to form a vector <math>X</math>, which is then linearly transformed to form a 2-dimensional random vector <math>Y=AX</math>. Find a matrix <math>A</math> so that the components of <math>Y</math> are independent random variables You must justify your answer.
+
 
+
:'''Click [[ECE_PhD_QE_CNSIP_2013_Problem1.2|here]] to view student [[ECE_PhD_QE_CNSIP_2013_Problem1.2|answers and discussions]]'''
+
----
+
'''Part 3.'''
+
 
+
Let <math>X</math> be an exponential random variable with parameter <math>\lambda</math>, so that <math>f_X(x)=\lambda{exp}(-\lambda{x})u(x)</math>. Find the variance of <math>X</math>. You must show all of your work.
+
 
+
:'''Click [[ECE_PhD_QE_CNSIP_2013_Problem1.3|here]] to view student [[ECE_PhD_QE_CNSIP_2013_Problem1.3|answers and discussions]]'''
+
----
+
'''Part 4.'''
+
 
+
Consider a sequence of independent random variables <math>X_1,X_2,...</math>, where <math>X_n</math> has pdf
+
 
+
<math>\begin{align}f_n(x)=&(1-\frac{1}{n})\frac{1}{\sqrt{2\pi}\sigma}exp[-\frac{1}{2\sigma^2}(x-\frac{n-1}{n}\sigma)^2]\\
+
&+\frac{1}{n}\sigma exp(-\sigma x)u(x)\end{align}</math>.
+
 
+
Does this sequence converge in the mean-square sense? ''Hint:'' Use the Cauchy criterion for mean-square convergence, which states that a sequence of random variables <math>X_1,X_2,...</math> converges in mean-square if and only if <math>E[|X_n-X_{n+m}|] \to 0</math> as <math>n \to \infty</math>, for every <math>m>0</math>.
+
  
:'''Click [[ECE_PhD_QE_CNSIP_2013_Problem1.4|here]] to view student [[ECE_PhD_QE_CNSIP_2013_Problem1.4|answers and discussions]]'''
 
 
----
 
----
 
[[ECE_PhD_Qualifying_Exams|Back to ECE Qualifying Exams (QE) page]]
 
[[ECE_PhD_Qualifying_Exams|Back to ECE Qualifying Exams (QE) page]]

Latest revision as of 21:52, 24 April 2017


ECE Ph.D. Qualifying Exam

Fields and Optics (FO)

Question 1: Statics 1

August 2013



Question 1

Alt text

Alt text

Solution

Click here to view student answers and discussions

Question 2

Alt text

Solution

Click here to view student answers and discussions

Question 3

Alt text

Alt text

Solution

Click here to view student answers and discussions

Back to ECE Qualifying Exams (QE) page

Alumni Liaison

Recent Math PhD now doing a post-doctorate at UC Riverside.

Kuei-Nuan Lin